there is a real number n such that numerical values of sum of the edge lengths and surface area of a cube with side length n are equal. find n?

Answers

Answer 1

Answer:

Given:

Sum of edge lengths = TSA of cube

there are 12 edges in a cube

[tex]12n = 6 {n}^{2} \\ 6 {n}^{2} - 12n = 0 \\ 6n(n - 2) = 0 \\ 6n = 0 \\ n = 0 \\ n - 2 = 0 \\ n = 2[/tex]

n ≠ 0 (since length can't be 0)

therefore n=2


Related Questions

(1 point) For each of the following pairs of groups G1, G2, determine the number of elements in the direct product G, x G2 of the given order (D6 is the dihedral group of order 12). G G2knumber of elements in G x G2 of order k Z16 Z19 4 U(14) U(13)6

Answers

The number of elements in the direct product are 2.

What is direct product of groups?

The direct product, often written as G x H, is an operation used in group theory, a branch of mathematics, to create a new group from two existing ones. This operation, one of several crucial direct product concepts in mathematics, is the group-theoretic equivalent of the Cartesian product of sets.

Let, the number of elements of order 1 in  [tex]Z_1_6[/tex]  and  [tex]Z_1_9[/tex]  is 1.

Number of elements of order 2 in [tex]Z_1_6[/tex]  and [tex]Z_1_9[/tex]  are 1 and 0 respectively.

Number of elements of order 4 in [tex]Z_1_6[/tex]  and  [tex]Z_1_9[/tex] are 2 and 0 respectively.

Consider the table of number of elements of order.

                           Number of elements of order

                              1         2          4

           [tex]Z_1_6[/tex]              1         1           2

          [tex]Z_1_9[/tex]               1         0          0

Let x ∈ [tex]Z_1_6[/tex] x [tex]Z_1_9[/tex]

O(x) = lcm (O(a), O(b))

where x = (a, b)

That is, a ∈ [tex]Z_1_6[/tex]  and  b ∈ [tex]Z_1_9[/tex]  

Here only (4, 1) satisfies it and [tex]Z_1_6[/tex] has 2 elements of order 4 and [tex]Z_1_9[/tex]  has one element of order 1.

So,  [tex]Z_1_6[/tex] x [tex]Z_1_9[/tex] has 2 x 1 = 2 elements of order 4.

Hence, the number of elements in the direct product are 2.

To know more about direct product of groups, click on the link

https://brainly.com/question/17012757

#SPJ4

Complete question:

Complete question is attached below.

point (9,1) and (7,R) are on a line with a slope of -3/4 what is R

Answers

The value of R is 2.5 or (5/2) . We can find the answer by using concepts of slope of line.

What is the slope of a line?

Slope of a line measures its steepness . Mathematically, its calculated as "rise over run".

When two points are given,

Slope of a line = (y2 - y1)/(x2 - x1)  

Here, y2 = R

          y1 = 1

         x2 = 7

          x1 = 9

Slope of a line = (R-1)/(7-9)

In the que, its given slope  = -3/4

So, (R-1)/(-2) = -3/4

      R = 5/2 or 2.5

Learn more about slope of a line using given link:
https://brainly.com/question/16949303

#SPJ1

Please help, problem/picture is attached

Answers

Answer:

Step-by-step explanation:

dudjdjd

Twenty-five samples of 100 items each were inspected when a process was considered to be operating satisfactorily. In the 25 samples, a total of 135 items were found to be defective.
What is an estimate of the proportion defective when the process is in control (to 3 decimals)?
What is the standard error of the proportion if samples of size 100 will be used for statistical process control (to 4 decimals)?
Compute the upper control limit and lower control limit for the control chart (to 4 decimals, if necessary)?

Answers

a) The estimate of the proportion of defectives when the process is in control is 0.054

b) The standard error of the proportion if the sample size is 100 is 0.0226.

c) The upper control limit is 0.1218 and the lower control limit is 0 (since LCL < 0 and p > 0, we can write LCL = 0).

What are the formulas for finding the estimate of the proportion, standard variation, and control limits?

1) The estimate of the proportion of success is

p = (number of success)/(total number of samples)

I.e., p = x/N

2) The standard deviation of the proportion of success is

[tex]\sigma_p = \sqrt{\frac{p(1-p)}{n} }[/tex]

3) The upper and lower control limits for a control chart are:

L.C.L = p - 3[tex]\sigma_p[/tex]

and U.C.L = p + 3[tex]\sigma_p[/tex]

Calculation:

It is given that, there are 25 samples of 100 items each.

So, the total number of items i.e., the total sample size,

N = 25 × 100 = 2500

In 25 samples, a total of 135 items were found to be defective.

So, the number of defectives x = 135

a) The estimate of the proportion of defectives is p = x/N

On substituting, we get

p = 135/2500 = 0.054

b) The standard error of the proportion if the sample of size 100 is calculated by

[tex]\sigma_p = \sqrt{\frac{p(1-p)}{n} }[/tex]

On substituting p = 0.054 and  n = 100, we get

[tex]\sigma_p = \sqrt{\frac{0.054(1-0.54)}{100} }[/tex]

    = 0.0226

c) The control limits for the control chart are:

Upper control limit =  p + 3[tex]\sigma_p[/tex]

⇒ U.C.L = 0.054 + 3(0.0226) = 0.054 + 0.0678 = 0.1218

Lower control limit = p - 3[tex]\sigma_p[/tex]

⇒ L.C.L = 0.054 - 3(0.0226) = 0.054 - 0.0678 = - 0.0138 ≈ 0

(Since we know that the lower control limit should not be a negative value, it is made equal to 0).

Learn more about an estimate of the proportion here:

https://brainly.com/question/23986522

#SPJ4

In how many ways can Tim draw a red or a green marble from a jar containing 20 red marbles and 4 green marbles?

Answers

The correct answer to how many different ways he can draw is 24.

Given the equation, proof it

Answers

Answer:

Step-by-step explanation:

I forgot to add the last reason is AAS (two angles one side are congruent = triangles are congruent)

S.

1.Given

2. AE = EC

3. <AEB = <CED

4. <CDE = <ABE

5. triangles congruent

R.

1. Given

2. Segment bisector theorem

3. Vertical angles theorem

4. PAI Theorem

Hope this helps!

Help Please...
Most adult medication doses are for a person weighing 150 pounds. For a 45-pound child, the adult dose should be multiplied by 0.3. If a child's dose of a decongestant is 9 milligrams, what is the adult dose? Please show your work.

Answers

The adult dose for the decongestant is 2.7 milligrams.

How to calculate the adult dose?

The information can be illustrated through an expression. Expression simply refers to the mathematical statements which have at least two terms which are related by an operator and contain either numbers, variables, or both. Addition, subtraction, multiplication, and division are all possible mathematical operations.

Since for a 45-pound child, the adult dose should be multiplied by 0.3, When the child's dose of a decongestant is 9 milligrams, the adult dose will be:

= Children dose × 0.3

= 9 × 0.3

= 2.7

Learn more about expressions on;

brainly.com/question/723406

#SPJ1

-3x+y=2
-x+y-4=0
solving a system of linear equations by substitution

Answers

The solution to the system of linear equations by substitution is (1, 5)

How to solve the system of linear equations by substitution

From the question, we have the following parameters that can be used in our computation:

-3x+y=2

-x+y-4=0

Rewrite as

-3x + y = 2

-x + y - 4 = 0

Make y the subject in -x + y - 4 = 0

So, we have

y = x + 4

Substitute y = x + 4 in -3x + y = 2

-3x + x + 4 = 2

Evaluate the like terms

-2x = -2

So, we have

x = 1

Substitute x = 1 in y = x + 4

y = 1 + 4

Evaluate

y = 5

Hence, the solution is  x = 1 and y = 5

Read more about system of equations at

https://brainly.com/question/13729904

#SPJ1

When practicing statistics in real life, it is not very important to check the necessary assumptions of a statistical procedure in order to effectively carry out and use the results. True False

Answers

The given statement is FALSE.

Given statement;

When practicing statistics in real life, it is not very important to check the necessary assumptions of a statistical procedure to effectively carry out and use the results.

The above-mentioned statement is FALSE.

→ We are aware that checking assumptions is crucial when applying statistics in daily life because failing to do so will prevent us from receiving fair answers to our statistical queries or drawing valid inferences. To use the good process and appropriate statistical distributions while utilizing statistics in daily life, it is crucial to examine assumptions.

To learn more about statistics click here:

brainly.com/question/23091366

#SPJ4

If the discriminant of an equation is negative, which of the following is true of the equation.

Answers

Equations have two complex solutions if their discriminant is negative. So, option A is the correct answer.

In algebra, the discriminant of any equation is a quantity that is used to find out about the nature of the roots using the formula = b²-4ac. It tells us if there are no solutions, one solution, or two.

Here, we have been told that the discriminant is negative.

From the formula, we can say that = D = b²-4ac < 0.

Here, if D<0, then x becomes an imaginary value. This means that the roots are going to be imaginary i.e., complex. Thus, the two roots that we get as a result of the formula are both going to be complex.

So, option A is the answer.

The complete that you might be looking for is given below.

If the discriminant of an equation is negative, which of the following is true of the equation

A. it has two complex solutions

B. it has one real solution

C. it had two real solutions

Learn more about the Discriminant on

https://brainly.com/question/15884086?referrer=searchResults

#SPJ4

Answer:

a.two complex solutions

Step-by-step explanation:

y <= -15x + 3000y <= 5xIn the xy plane, if a point with coordinates (a,b) lies in the solution set of the system of inequalities above, what is the maximum possible value for b?

Answers

B can have a maximum value of 750.

Given info,

y ≤ - 15x + 3000

y ≤ 5x

What is the highest possible value of b in the XY plane if a point with the coordinates (a,b) is found in the system of inequalities above?

The maximum value of b occurs when taking equality.

⇒ - 15x + 3000 = 5x

⇒ - 20x = - 3000

⇒ - 20x = - 3000

⇒ x = 150

Therefore, y = 5 * 150

                    = 750

Hence, the maximum possible value of b = 750

To learn more about equality click here:

brainly.com/question/9070018

#SPJ4

construct a recursive binary search algorithm by putting the following steps in the correct order.

Answers

CASE1: If target is equal to middle, then return mid.

CASE2:If target is less than middle i.e., target<A[mid],we discard all the elements in the right search space including mid element. Now our new high would be mid-1 while 'low' remains as it is.

CASE3:If the target element is greater than middle i.e., target>A[mid],we discard all the elements in the left search space including mid element. Now our new low would be mid+1 while 'high' remains as it is.

What is Recursive Algorithm?

Recursive algorithm, a function calls itself again and again till the base condition(stopping condition) is satisfied.

Let us track the search space by using two index start and end. Initialy low=0 and high=n-1(as initialy whole array is search space).At each step, we find mid value in the search space and compare it with target value. There are three cases possible:

CASE1: If target is equal to middle, then return mid.

CASE2:If target is less than middle i.e., target<A[mid],we discard all the elements in the right search space including mid element. Now our new high would be mid-1 while 'low' remains as it is.

CASE3:If the target element is greater than middle i.e., target>A[mid],we discard all the elements in the left search space including mid element. Now our new low would be mid+1 while 'high' remains as it is.

Recursive implementation of binary search algorithm, in the method binarySearch(), follows almost the same logic as iterative version, except for a couple of differences.The first difference is that the while loop is replaced by a recursive call back to the same method with the new values of low and high passed to the next recursive invocation along with "Array" and "key" or target element.The second difference is that instead of returning false when the while loop exits in the iterative version, in case of the recursive version, the condition of low > high is checked at the beginning of the next level of recursion and acts as the boundary condition for stopping further recursive calls by returning false.Also, note that the recursive invocations of binarySearch() return back the search result up the recursive call stack so that true or false return value is passed back up the call stack without any further processing.

To learn more about binarySearch:

https://brainly.com/question/29487950

#SPJ4

True or False: The range of sine is...
True
False

Answers

It is false that the range of the sine function is 0 ≤ ∅ ≤ π

How to determine the true statement?

From the question, we have the following parameters that can be used in our computation:

Function: sine function

Also from the question, we have

Range: 0 ≤ ∅ ≤ π

As a general rule, we have:

The minimum value of a sine function is -1The maximum value of a sine function is 1

This means that the range of the sine function is from [-1, 1].

This can also be represented as -1 ≤ ∅ ≤ 1

Hence. the given statement about the sine function is false

Read more about sine functions at

https://brainly.com/question/21902442

#SPJ1

Answer:

The answer is false.

Step-by-step explanation:

The range of the sine function is NOT 0 ≤ ∅ ≤ π

Point A is the incenter of triangle DEF.
Which must be true? Select three options

A.Point A is the center of the circle that passes through points D, E, and F.
B.Point A is the center of the circle that passes through points X, Y, and Z.
C.ZA≅YA
D.EA≅FA
E.AX≅AY

Answers

The three options that describe the in-center of the given triangle are;

B. Point A is the center of the circle that passes through points X, Y, and Z.

C. ZA ≅ YA

E. AX ≅ AY

What is the in center of the triangle?

The incenter of a triangle is defined as the point of intersection point of all the three interior angle bisectors of the given triangle. This means that, it can also be said to be the point where all the internal angle bisectors of the triangle cross each other.

Now, we are told that Point A is the incenter of triangle DEF. From the definition above, we can deduce that;

An inscribed circle can be drawn with center A passing through points X, Y and Z.

Similarly, since it passes through X, Y and Z, then it means that ZA≅YA, AX≅AY.

Read more about Triangle In-center at; https://brainly.com/question/1831482

#SPJ1

it is possible for two sets of data to have the same regression equation but different correlations.

Answers

Yes, it is possible for two sets of data to have the same regression equation but different correlations.

What is regression and correlation?

Correlation quantifies the strength of the linear relationship between a pair of variables, whereas regression expresses the relationship in the form of an equation.

The use of correlation analysis allows for the quantification of relationships between two continuous variables, such as those between two independent variables or between a dependent and independent variable.

Regression analysis is the process of determining how one or more variables and the outcome variable relate to one another. Risk factors, co-founders, and the outcome variable are all referred to as predictors or independent variables. The outcome variable is also referred to as the dependent or response variable. Regression analysis displays the dependent variable as "y" and the independent variables as "x".

To learn more about regression and correlation:

brainly.com/question/4219149

#SPJ4

One of the 55 students are selected at random.written as a fraction what is the probability

Answers

If one of the 55 students is selected at random, the probability of this occurring can be written as a fraction by dividing the number of selected students by the total number of students. In this case, the probability would be 1/55.

The UGA Journalism Department is interesting in determining if there is a significant difference in the number of hours, on average per week, that UGA males and UGA female students spend reading any online or printed newspaper. Randomly selected students at UGA were asked how many hours per week they read any online or printed newspaper, their age, and gender. The 95% confidence interval for reading newspapers on μF-urn is (-1.1, 1.3). 1) Interpret this confidence interval. Because 0 is not in this interval, we cannot conclude that there is a difference in newspaper reading, on average, for females and males. Because 0 is not in this interval, we can conclude that there is a difference in newspaper reading, on average, for females and males. Because 0 is in this interval, we cannot conclude that there is a difference in newspaper reading, on average, for females and males. Because 0 is in this interval, we can conclude that there is a difference in newspaper reading, on average, for females and males.

Answers

The option C, “because 0 is in this interval, we cannot conclude that there is a difference in newspaper reading, on average, for females and males” is correct.

In the given question, the UGA Journalism Department is interesting in determining if there is a significant difference in the number of hours, on average per week, that UGA males and UGA female students spend reading any online or printed newspaper.

Randomly selected students at UGA were asked how many hours per week they read any online or printed newspaper, their age, and gender. The 95% confidence interval for reading newspapers on μ(F)-μ(M) is (-1.1, 1.3).

1) We have to interpret this confidence interval. The given options are:

• Because 0 is not in this interval, we cannot conclude that there is a difference in newspaper reading, on average, for females and males.

• Because 0 is not in this interval, we can conclude that there is a difference in newspaper reading, on average, for females and males.

• Because 0 is in this interval, we cannot conclude that there is a difference in newspaper reading, on average, for females and males.

• Because 0 is in this interval, we can conclude that there is a difference in newspaper reading, on average, for females and males.

Since there interval was given.

The given interval for the 95% confidence interval for reading newspapers on μ(F)-μ(M) is (-1.1, 1.3).

Since the value of interval is between -1.1 and 1.3. So 0 is in this interval. So we cannot conclude that there is a difference in newspaper reading, on average, for females and male.

So the option C, “because 0 is in this interval, we cannot conclude that there is a difference in newspaper reading, on average, for females and males” is correct.

To learn more about confidence interval link is here

brainly.com/question/29657446

#SPJ4

Tanya has a collection of 52 rocks that she wants to put into display cases. Each display case can hold 8 rocks. Tanya divides to find how many cases
she needs.
52÷8=614
How many display cases will Tanya need?

A. 4

B. 6

C. 7

D. 8

E. 10

Answers

The answer would be (B


A 3-gallon container of window cleaner costs $17.76. What is the price per quart?

Answers

Answer: $1.48

Step-by-step explanation:

find how much each gallon cost

1. 17.76/3=5.92

Divide by 4 because 4 quarts equal a gallon

2. 5.92/4=1.48

Therefore each quart equals $1.48,

Hope this helps!

Which of the following is true when the discriminant of a quadratic equation is zero?

Answers

When an equation's discriminant is zero, the following is the given statement that is correct: There is just one viable solution.

Having the form ax2 + bx + c = 0, quadratic equations are second-degree algebraic expressions. The name "Quadratic" is derived from the word "Quad" which means square. A quadratic equation is an "equation of degree 2," to put it another way. A quadratic equation is employed in numerous situations.

There are a maximum of two solutions for x in the second-degree quadratic equations. These two solutions for x are denoted as (α, β) and are also known as the roots of quadratic equations.

Three instances of discrimination exist.The equation has two real solutions if the discriminant is strictly bigger than zero.There is just one real solution if the discriminant equals zero.Additionally, if the discriminant is strictly less than zero, there are no real solutions to the equation, meaning that both of its solutions are complex.

Learn more about quadratic equations here:

https://brainly.com/question/28038123

#SPJ4

3. What is the power output of an engine that does 60,000 J of work in 10 s?

Answers

the answer to the problem is 6 KILA watts

Important Formula:

[tex]p=\dfrac{w}{t}[/tex]

Where p is power(measured in watts), w is work(measured in joules), and t is time(measured in seconds).

[tex]w=60,000J[/tex]

[tex]t=10s[/tex]

[tex]p=?[/tex]

____________

[tex]p=\dfrac{w}{t}[/tex]

____________

[tex]p=\dfrac{60,000}{10}[/tex]

____________

[tex]\fbox{p = 6,000 watts}[/tex]

find the absolute maximum and absolute minimum of f (x, y) among points in the triangle with vertices (0, 0), (1, 0), and (1, 1).

Answers

The absolute maximum and absolute minimum of f(x,y) = 3 + xy -x - 2y is f(0,0) and f(1,1) respectively.

The absolute maximum point is a point where the function obtains its greatest possible value.

And Absolute minimum is a point where the function obtains its least possible value. This is the smallest value that a mathematical function can have over its entire curve.

Given equation is, f(x,y) = 3 + xy - x - 2y

we will check absolute maximum and absolute minimum of f(x,y) at the points (0,0), (1,0), (1,1).  

f(0,0) = 3 + (0)(0) - 0 - 2(0) = 3

f(1,0) = 3 + (1)(0) - 1 - 2 (0) = 2

f(1,1) = 3 + (1)(1) - 1 - 2(1) = 1

Therefore we get absolute maximum at f(0,0) and absolute minimum at f(1,1).

An absolute minimum also called a global minimum, occurs when a point of the function is lower any other point on the function within the function's domain. A local minimum also called relative minimum occurs when a point is lower than the points surrounding it.

Given question is incomplete. Complete question is:

find the absolute maximum and absolute minimum of f (x, y) = 3 + xy - x -2y  among points in the triangle with vertices (0, 0), (1, 0), and (1, 1).

To know more about maximum here

https://brainly.com/question/13176730

#SPJ4

TRUE/FALSE. incidence of multiple chronic conditions is connected to frailty, a condition marked by a lack of resilience to physiological changes and an elevated risk of poor health outcomes.

Answers

The given

Incidence of multiple chronic conditions is connected to frailty, a condition marked by a lack of resilience to physiological changes and an elevated risk of poor health outcomes.

It is TRUE.

"Information available from the question:"

In the question:

Incidence of multiple chronic conditions is connected to frailty, a condition marked by a lack of resilience to physiological changes and an elevated risk of poor health outcomes.

It is TRUE / FALSE

Now, According to the question:

Let's know:

What is Frailty?

Frailty is most often defined as an aging-related syndrome of physiological decline, characterized by marked vulnerability to adverse health outcomes.

Incidence of multiple chronic conditions is connected to frailty, a condition marked by a lack of resilience to physiological changes and an elevated risk of poor health outcomes.

Hence, The above statement is TRUE.

Learn more about Frailty at:

https://brainly.com/question/28287310

#SPJ4

Give an example of odd function and also of even function ​

Answers

Therefore, the example of the polynomial function f(x)=[tex]x^{2} +x^{4} +x^{6}[/tex] is even and the polynomial function f(x)=[tex]x+x^{3} +x^{5}[/tex] is odd

Function is what?

A mathematical function from one set X to another gives each element of X exactly one element of the other. The sets X and Y, respectively, represent the function's domain and codomain.

Here,

The polynomial function f(x)=[tex]x^{2} +x^{4} +x^{6}[/tex] is even.

The polynomial function f(x)=[tex]x+x^{3} +x^{5}[/tex] is odd

As we can see,

The function

f(x)=[tex]x^{2} +x^{4} +x^{6}[/tex]

when we put

f(-x)=[tex](-x)^{2} +(-x)^{4} +(-x)^{6}[/tex]

f(-x)=[tex]x^{2} +x^{4} +x^{6}[/tex]

Thus f(x)=f(-x)

Therefore it is even.

The function

f(x)=[tex]x+x^{3} +x^{5}[/tex]

f(-x)=[tex](-x)+(-x)^{3} +(-x)^{5}[/tex]

f(-x)=[tex]-x-x^{3} -x^{5}[/tex]

Thus f(x)≠f(-x)

Therefore it is odd.

Therefore, the example of the polynomial function f(x)=[tex]x^{2} +x^{4} +x^{6}[/tex] is even and the polynomial function f(x)=[tex]x+x^{3} +x^{5}[/tex] is odd

To know more about function , visit

brainly.com/question/5975436

#SPJ4

A tile is randomly selected from a bag that contains 10 tiles. The tiles are either blue, green, red, or yellow. After the color is recorded, the tile is placed back in the bag. A tile is pulled 100 times, of which 24 are blue tiles, 12 are green tiles, and 12 are red tiles.

Answers

The number of tiles in the bag of 10 that are yellow in colour are; 5 tiles

How to find the probability of selection?

We are given;

Total number of tiles in bag = 10 tiles

Now, we are told that in the bag, the tiles are either blue, green, red, or yellow.

Now, after selecting a tile, we are told that it is put back and as such we have after 100 selections that we have 24 blue tiles, 12 green tiles and 12 red tiles. Thus;

Number that will be yellow = 100 - 24 - 12 - 12 = 52 yellow tiles

Thus, it means that the probability of selecting a yellow tile is;

52/100

Thus, number of tiles out of the 10 in the bag that are yellow =

(52/100) * 10 = 5.2

Approximately 5 yellow tiles

Read more about probability of selection at; https://brainly.com/question/251701

#SPJ1

Complete question is;

How many yellow tiles are most likely in the bag? Enter the answer in the box. A tile is randomly selected from a bag that contains 10 tiles. The tiles are either blue, green, red, or yellow. After the color is recorded, the tile is placed back in the bag. A tile is pulled 100 times, of which 24 are blue tiles, 12 are green tiles, and 12 are red tiles.​

evaluate the expression: v ⋅ w given the vectors: r = <8, 8, -6>; v = <3, -8, -3>; w = <-4, -2, -6> (2 points)

Answers

The product of the vectors v and w is gotten as; v ⋅ w = 22

What is the product of the vectors?

We are given the vectors;

r = <8, 8, -6>

v = <3, -8, -3>

w = <-4, -2, -6>

We want to evaluate v.w which is a product of both vectors.

The formula will be;

v.w = (v_x * w_x) + (v_y * w_y) + (v_z * w_z)

Plugging in the values and solving gives;

v.w = (3 * -4) + (-8 * -2) + (-3 * -6)

v.w = -12 + 16 + 18

v.w = 22

We can conclude that the vector expressions has a product of 22

Read more about Vector Products at; https://brainly.com/question/9131049

#SPJ1

A pharmacy has determined that a healthy person should receive 70 units of
proteins, 100 units of carbohydrates and 20 units of fat daily. If the store carries
the six types of health food with their ingredients as shown in the table below,
what blend of foods satisfies the requirements at minimum cost to the
pharmacy? Make a mathematical model for the given problem.
Foods Protein units Carbohydrates
units
Fat units Cost per unit
A 20 50 4 2
B 30 30 9 3
C 40 20 11 5
D 40 25 10 6
E 45 50 9 8
F 30 20 10 8

Answers

optimal solution is arrived with value of variables as :

x1=0.9091,x2=1.8182,x3=0,x4=0,x5=0,x6=0

minimize cost:

z=2x₁+3x₂+5x₃+6x₄+8x₅+8x₆=2x

subject to:

20x₁+30x₂+40x₃+40x₄+45x₅+30x₆ ≥70 : amount of protein

50x₁+30x₂+20x₃+25x₄+50x₅+20x₆ ≥100 : amount of carbohydrate

4x₁+9x₂+11x₃+10x₄+9x₅+10x₆ ≥20 : amount of fat

where x₁, x₂, x₃, x₄, x₅, x₆ are units of 6 foods

solution using Simplex method:

After introducing artificial variables:

Min Z=2x₁+3x₂+5x₃+6x₄+8x₅+8x₆+0S₁+0S₂+0S₃+MA₁+MA₂+MA₃

subject to

20x₁+30x₂+40x₃+40x₄+45x₅+30x₆-S₁+A₁ ≥ 70

50x₁+30x₂+20x₃+25x₄+50x₅+20x₆-S₂+A₂≥ 100

4x₁+9x₂+11x₃+10x₄+9x₅+10x₆-S₃+A₃=20

and x₁,x₂,x₃,x₄,x₅,x₆,A₁,A₂,A₃≥0

Refer to the image for Z

Since all Zj-Cj≤0

Hence, optimal solution is arrived with value of variables as :

x1=0.9091,x2=1.8182,x3=0,x4=0,x5=0,x6=0

Min Z=7.2727

Learn more about Simplex Method at:

https://brainly.com/question/14410494

#SPJ1

Let E be the event that a corn crop has an infestation of ear worms, and let B be the event that a corn crop has an infestation of corn borers. Suppose that P(E) = 0.24, P(B) = 0.16, and P(E and B) = 0.13. Find the probability that a corn crop has either an ear worm infestation, a corn borer infestation, or both.0.530.130.60.27

Answers

The probability that a corn crop has either an ear worm infestation, a corn borer infestation, or both is 0.27.

What is probability?

The area of mathematics known as probability deals with numerical descriptions of how likely it is for an event to happen or for a proposition to be true. A number between 0 and 1 is the probability of an event, where, roughly speaking, 0 denotes the event's impossibility and 1 denotes its certainty.

We have to find the probability that a corn crop has either an ear worm infestation, a corn borer infestation, or both.

Since,

P(E or B) = P(E) + P(B) - P(E and B)

Given,

P(E) = 0.24

P(B) = 0.16

P(E and B) = 0.13

Hence,

P(E or B) = 0.24 + 0.16 - 013

P(E or B) = 0.27

Option D is correct.

Hence, the probability that a corn crop has either an ear worm infestation, a corn borer infestation, or both is 0.27.

To know more about probability, click on the link

https://brainly.com/question/24756209

#SPJ4

If X is a metric space, and fn:X→R (n∈N) is a sequence of functions, then fn converges pointwise to f if for every x∈X one has limn→[infinity]fn(x)=f(x).

Answers

|f(x) − f n(x)| = lim m→∞

|f m(x) − f n(x)| ≤ ϵ /2< ϵ

Converges Uniformly

A sequence of functions f n(x); n = 1, 2, 3,…. Is said to be uniformly convergent to f for a set E of values of x, if for each ε > 0, a positive integer N exists such that |f n(x) – f(x)| < ε for n ≥ N and x ∈ E.

A sequence (f n) of functions f n : A → R is uniformly Cauchy

on A if for every ϵ > 0 there exists N ∈ N such that

m, n > N implies that |f m(x) − f n(x)| < ϵ for all x ∈ A.

The key part of the following proof is the argument to show that a pointwise

convergent, uniformly Cauchy sequence converges uniformly.

A sequence (f n) of functions f n : A → R converges uniformly on

A if and only if it is uniformly Cauchy on A.

Suppose that ( f n) converges uniformly to f on A. Then, given ϵ > 0, there

exists N ∈ N such that

|f n(x) − f(x)| < ϵ/2

for all x ∈ A if n > N.

It follows that if m, n > N then

|f m(x) − f n(x)| ≤ |f m(x) − f(x)| + |f(x) − f n(x)| < ϵ for all x ∈ A,

which shows that (f n) is uniformly Cauchy.

Conversely, suppose that (f n) is uniformly Cauchy. Then for each x ∈ A, the

real sequence (f n(x)) is Cauchy, so it converges by the completeness of R. We

define f : A → R by

f(x) = limn→∞

f n(x),

and then f n → f pointwise.

To prove that f n → f uniformly, let ϵ > 0. Since (f n) is uniformly Cauchy,

can choose N ∈ N (depending only on ϵ) such that

|f m(x) − f n(x)| < ϵ /2

for all x ∈ A if m, n > N.

Let n > N and x ∈ A. Then for every m > N we have

|f n(x) − f(x)| ≤ |f n(x) − f m(x)| + |f m(x) − f(x)| < ϵ /2 + |f m(x) − f(x)|.

Since f m(x) → f(x) as m → ∞, we can choose m > N (depending on x, but it

doesn’t matter since m doesn’t appear in the final result) such that

|f m(x) − f(x)| < ϵ/2.

It follows that if n > N, then

|f n(x) − f(x)| < ϵ for all x ∈ A,

Then proves that f n → f uniformly

Alternatively, we can take the limit as m → ∞ in the Cauchy condition to get

for all x ∈ A and n > N that

|f(x) − f n(x)| = lim m→∞

|f m(x) − f n(x)| ≤ ϵ /2< ϵ

To learn more about Converges Uniformly visit:

brainly.com/question/29736068

#SPJ4

A sprinter completed a 100 m run with a time of 12.31 seconds.
Her time,
t
, has been rounded to the nearest hundredth of a second.
Write the error interval for
t
in the form
a

t
<
b
.

Answers

The error interval for the time spent to complete the race is 12.305 ≤ t < 12.315

How to determine the error interval for the time spent to complete the race

From the question, we have the following parameters that can be used in our computation:

Distance = 100 m

Time = 12.31 seconds

From the question, we understand that

Time = rounded to the nearest hundredth of a second.

The highest number that can be rounded to 12.31 to the nearest hundredth is 12.314

While the least is 12.305

When represented as an interval, we have

12.305 ≤ t < 12.314

Add/subtract the error

So, we have

12.305 - 0.001 ≤ t < 12.314 + 0.001

Evaluate

12.305 ≤ t < 12.315

Hence, the error interval is 12.305 ≤ t < 12.315

Read more about error margin at

https://brainly.com/question/25613341

#SPJ1

Other Questions
Help please- i hate Spanish the three types of factors that influence the breakup of relationships as identified by george levinger are attraction, the alternatives one possesses, and the . Metaethics is largely about studying dr. washington believes that mykel's anxiety is related to his inhibited temperament. which of the following is she most likely to recommend as treatment for mykel? ergonomics is a multidisciplinary science that seeks to conform the workplace and all of its physiological aspects to the worker. judi has recently been experiencing lower back pain. she has had diagnostic tests, but they did not suggest a definite cause. after trying several medications to no avail, she is considering alternative remedies. which of the following is most likely to help with her back pain? __________ can be used to attack databases.A. Buffer overflowsB. SQL injectionC. Buffer injectionD. Input validation in a highly stressful job situation, mathew works hard because he knows that he'll get a bonus if he meets his goals. ginny works hard because when she meets her goals she feels a sense of accomplishment . who is most likely to perform better? group of answer choices A contract to do something that is prohibited by statute isenforceable if the parties are aware of the prohibition.enforceable if the parties are ignorant of the prohibition.enforceable if the contract does not negatively affect society.unenforceable. A service account is created by the system or an application and cannot be used to log in to the system.Which of the following methods can be used to verify that a service account cannot login to the system?a. View the entry for the service account in /etc/passwd and look for /sbin/nologin.b. View the entry for the service account in /etc/shadow and look for /sbin/nologin.c. View the ACLs for /bin/login to ensure that the service account is not listed.d. Verify that file and directory permissions have been removed for the service account to the /boot partition. which option proves the following statement by contradiction? for all real numbers r and s, if r is rational and s is irrational, then r 2s is irrational. proof (by contradiction): suppose not. that is, suppose there are real numbers r and s such that r is rational and s is irrational and r 2s is rational. [we must show that this supposition leads logically to a contradiction.] by definition of rational, Which part of Article II, Section 2 of the US Constitution gives presidents the power to fulfill one of their roles as chief diplomat?He shall nominate, and by and with the Advice and Consent of the Senate, shall appoint Ambassadors.He shall nominate, and by and with the Advice and Consent of the Senate, shall appoint . . . Judges of the supreme Court.He shall have Power, by and with the Advice and Consent of the Senate, to make Treaties, provided two thirds of the Senators present concur.The President shall be Commander in Chief of the Army and Navy of the United States, and of the Militia of the several States, when called into the actual Service of the United States. Match the activity with its corresponding step in the personal selling process.1.Prospecting2.Preapproach3.Presentation4.Close5.Follow-up for this reaction it was observed that the first three half-lives were 12.5 h, 25 h, and 50 h when a you are forecasting incremental free cash flows for daily enterprises. based on the information in question 1 and 2, what are the incremental free cash flows associated with the new machine in year 0? the state of texas has one of the highest sales tax rates in the nation at 6.25 percent. this tax can have another added to it to help fund county and local governments. You turn on your Windows 7 computer and see the system display POST messages. Then the screen turns blue with no text. Which of the following items could be the source of the problem?a. The video cardb. The monitorc. Windowsd. Microsoft Word software installed on the system the weight, in grams, of a population of bacteria at time t hours is modeled by the function w, the solution to a logistic differential equation. selected values of w and its first and second derivatives are shown in the table above. which of the following statements is true? a wire has a diameter of 0.852cm and is 8.49m long. when connected to a 15.5v voltage source, the wire carries a 75.3a current. what is the resistivity, in ohm meters, of the wire? What Is Choking? Causes, Treatment, and Prevention